Proving...

Does it take more energy to get to Venus or to Mars?

If the heap is initialized for security, then why is the stack uninitialized?

Was a professor correct to chastise me for writing "Prof. X" rather than "Professor X"?

Is it my responsibility to learn a new technology in my own time my employer wants to implement?

Implement the Thanos sorting algorithm

What does "Its cash flow is deeply negative" mean?

When airplanes disconnect from a tanker during air to air refueling, why do they bank so sharply to the right?

Would this house-rule that treats advantage as a +1 to the roll instead (and disadvantage as -1) and allows them to stack be balanced?

How to make a variable always equal to the result of some calculations?

What's the point of interval inversion?

How can I open an app using Terminal?

Inappropriate reference requests from Journal reviewers

Are there languages with no euphemisms?

Why do professional authors make "consistency" mistakes? And how to avoid them?

What is the difference between "behavior" and "behaviour"?

How do I get the green key off the shelf in the Dobby level of Lego Harry Potter 2?

Rotate a column

How to safely derail a train during transit?

Is it safe to use c_str() on a temporary string?

Why didn't Theresa May consult with Parliament before negotiating a deal with the EU?

How can I quit an app using Terminal?

Term for the "extreme-extension" version of a straw man fallacy?

Shade part of a Venn diagram

Can the Reverse Gravity spell affect the Meteor Swarm spell?



Proving $left|begin{smallmatrix}1&1&1\a&b&c\a^3&b^3&c^3end{smallmatrix}right|=(b-a)(c-b)(c-a)(a+b+c)$



The Next CEO of Stack OverflowWhy is $left( begin{smallmatrix} x & y \ y & t \ end{smallmatrix} right)$ orthogonally similar to this?Finding $B,C$ such that $Bleft[begin{smallmatrix}1&2\4&8end{smallmatrix}right]C=left[begin{smallmatrix}1&0\0&0end{smallmatrix}right]$Minimal polynomial of $A := left(begin{smallmatrix} 7 & -2 & 1 \ -2 & 10 & -2 \ 1 & -2 & 7 end{smallmatrix}right)$Solve for A. $Bigl[begin{smallmatrix}9&9\-9&0end{smallmatrix}Bigr]=4A-Bigl[begin{smallmatrix}2&-2\0&2end{smallmatrix}Bigr]A$$begin{vmatrix} 1 & a &bc \ 1& b & ac\ 1&c & ab end{vmatrix}=begin{vmatrix} 1 & a &a^2 \ 1& b&b^2 \ 1& b & c^2 end{vmatrix}$Calculate the determinant $left|begin{smallmatrix} a&b&c&d\ b&a&d&c\ c&d&a&b\d&c&b&aend{smallmatrix}right|$Prove $left|begin{smallmatrix} sin^2x&cot x&1\ sin^2y&cot y&1\ sin^2z&cot z&1 end{smallmatrix}right|=0$ if $x+y+z=pi$Prove that $begin{vmatrix} xa&yb&zc\ yc&za&xb\ zb&xc&ya\ end{vmatrix}=xyzbegin{vmatrix} a&b&c\ c&a&b\ b&c&a\ end{vmatrix}$ if $x+y+z=0$Without expanding, show that $left| begin{smallmatrix} 3&4&5 \ 15&21&26 \ 21&29&36 \ end{smallmatrix}right|=0$Showing $P^TP=I_n-frac1n11^T$ if $left(begin{smallmatrix}frac1{sqrt n}&cdots&frac1{sqrt n}\&Pend{smallmatrix}right)$ is orthogonal












3












$begingroup$



Prove that$$begin{vmatrix}1&1&1\a&b&c\a^3&b^3&c^3end{vmatrix}=(b-a)(c-b)(c-a)(a+b+c)$$




My attempt:



$$begin{align}begin{vmatrix}1&1&1\a&b&c\a^3&b^3&c^3end{vmatrix}&=begin{vmatrix}0&1&0\a-b&b&c-b\a^3-b^3&b^3&c^3-b^3end{vmatrix}\&=begin{vmatrix}c-b&a-b\c^3-b^3&a^3-b^3end{vmatrix}\&=(c-b)(a-b)begin{vmatrix}1&1\c^2+cb+b^2&a^2+ab+b^2end{vmatrix}\&=(c-b)(a-b)[(a^2+ab)-(c^2+cb)]\end{align}$$



Where did I go wrong?










share|cite|improve this question











$endgroup$








  • 1




    $begingroup$
    check out the more general vandermonde determinant that might be useful.
    $endgroup$
    – Bijayan Ray
    Mar 16 at 9:05










  • $begingroup$
    try manually finding det through the 1st row, then its 8th grade algebra technique.
    $endgroup$
    – MotherLand
    Mar 16 at 9:20
















3












$begingroup$



Prove that$$begin{vmatrix}1&1&1\a&b&c\a^3&b^3&c^3end{vmatrix}=(b-a)(c-b)(c-a)(a+b+c)$$




My attempt:



$$begin{align}begin{vmatrix}1&1&1\a&b&c\a^3&b^3&c^3end{vmatrix}&=begin{vmatrix}0&1&0\a-b&b&c-b\a^3-b^3&b^3&c^3-b^3end{vmatrix}\&=begin{vmatrix}c-b&a-b\c^3-b^3&a^3-b^3end{vmatrix}\&=(c-b)(a-b)begin{vmatrix}1&1\c^2+cb+b^2&a^2+ab+b^2end{vmatrix}\&=(c-b)(a-b)[(a^2+ab)-(c^2+cb)]\end{align}$$



Where did I go wrong?










share|cite|improve this question











$endgroup$








  • 1




    $begingroup$
    check out the more general vandermonde determinant that might be useful.
    $endgroup$
    – Bijayan Ray
    Mar 16 at 9:05










  • $begingroup$
    try manually finding det through the 1st row, then its 8th grade algebra technique.
    $endgroup$
    – MotherLand
    Mar 16 at 9:20














3












3








3





$begingroup$



Prove that$$begin{vmatrix}1&1&1\a&b&c\a^3&b^3&c^3end{vmatrix}=(b-a)(c-b)(c-a)(a+b+c)$$




My attempt:



$$begin{align}begin{vmatrix}1&1&1\a&b&c\a^3&b^3&c^3end{vmatrix}&=begin{vmatrix}0&1&0\a-b&b&c-b\a^3-b^3&b^3&c^3-b^3end{vmatrix}\&=begin{vmatrix}c-b&a-b\c^3-b^3&a^3-b^3end{vmatrix}\&=(c-b)(a-b)begin{vmatrix}1&1\c^2+cb+b^2&a^2+ab+b^2end{vmatrix}\&=(c-b)(a-b)[(a^2+ab)-(c^2+cb)]\end{align}$$



Where did I go wrong?










share|cite|improve this question











$endgroup$





Prove that$$begin{vmatrix}1&1&1\a&b&c\a^3&b^3&c^3end{vmatrix}=(b-a)(c-b)(c-a)(a+b+c)$$




My attempt:



$$begin{align}begin{vmatrix}1&1&1\a&b&c\a^3&b^3&c^3end{vmatrix}&=begin{vmatrix}0&1&0\a-b&b&c-b\a^3-b^3&b^3&c^3-b^3end{vmatrix}\&=begin{vmatrix}c-b&a-b\c^3-b^3&a^3-b^3end{vmatrix}\&=(c-b)(a-b)begin{vmatrix}1&1\c^2+cb+b^2&a^2+ab+b^2end{vmatrix}\&=(c-b)(a-b)[(a^2+ab)-(c^2+cb)]\end{align}$$



Where did I go wrong?







linear-algebra matrices determinant






share|cite|improve this question















share|cite|improve this question













share|cite|improve this question




share|cite|improve this question








edited Mar 16 at 9:17









Rodrigo de Azevedo

13.2k41960




13.2k41960










asked Mar 16 at 8:36









DavidDavid

644




644








  • 1




    $begingroup$
    check out the more general vandermonde determinant that might be useful.
    $endgroup$
    – Bijayan Ray
    Mar 16 at 9:05










  • $begingroup$
    try manually finding det through the 1st row, then its 8th grade algebra technique.
    $endgroup$
    – MotherLand
    Mar 16 at 9:20














  • 1




    $begingroup$
    check out the more general vandermonde determinant that might be useful.
    $endgroup$
    – Bijayan Ray
    Mar 16 at 9:05










  • $begingroup$
    try manually finding det through the 1st row, then its 8th grade algebra technique.
    $endgroup$
    – MotherLand
    Mar 16 at 9:20








1




1




$begingroup$
check out the more general vandermonde determinant that might be useful.
$endgroup$
– Bijayan Ray
Mar 16 at 9:05




$begingroup$
check out the more general vandermonde determinant that might be useful.
$endgroup$
– Bijayan Ray
Mar 16 at 9:05












$begingroup$
try manually finding det through the 1st row, then its 8th grade algebra technique.
$endgroup$
– MotherLand
Mar 16 at 9:20




$begingroup$
try manually finding det through the 1st row, then its 8th grade algebra technique.
$endgroup$
– MotherLand
Mar 16 at 9:20










6 Answers
6






active

oldest

votes


















4












$begingroup$

Note that $(a^2+ab)-(c^2+cb)=(a-c)(a+c)+b(a-c)=(a-c)(a+b+c)$






share|cite|improve this answer









$endgroup$





















    2












    $begingroup$

    Given a $4times 4$ Vandermonde matrix
    $$
    left[begin{matrix}
    color{red}1&color{red}1&color{red}1&1\color{red}a&color{red}b&color{red}c&d\ a^2&b^2&c^2&color{blue}{d^2}\color{red}{a^3}&color{red}{b^3}&color{red}{c^3}&d^3
    end{matrix}right],
    $$
    note that the desired determinant is the minor of $d^2$. Considering Laplace expansion, it appears as a minus(-) coefficient of $d^2$ in the Vandermonde determinant
    $$begin{align*}
    &color{red}{(b-a)(c-a)(c-b)}(d-a)(d-b)(d-c)\=&color{red}{(b-a)(c-a)(c-b)}(d^3-color{blue}{(a+b+c)}d^2+cdots).
    end{align*}$$
    Thus we obtain the desired determinant
    $$color{red}{(b-a)(c-a)(c-b)}color{blue}{(a+b+c)}.
    $$






    share|cite|improve this answer











    $endgroup$













    • $begingroup$
      [+1] very interesting.
      $endgroup$
      – Jean Marie
      Mar 16 at 23:50










    • $begingroup$
      I just gave another way to consider this formula.
      $endgroup$
      – Jean Marie
      Mar 17 at 13:34



















    1












    $begingroup$

    $$begin{vmatrix}1&1\c^2+cb+b^2&a^2+ab+b^2end{vmatrix}$$



    $$=begin{vmatrix}1&1-1\c^2+cb+b^2&a^2+ab+b^2-(c^2+cb+b^2)end{vmatrix}$$



    $$=begin{vmatrix}1&0\c^2+cb+b^2&a^2-c^2+b(a-c)end{vmatrix}$$



    $$=a^2-c^2+b(a-c)$$



    $$=(a-c)(a+c+b)$$






    share|cite|improve this answer









    $endgroup$





















      1












      $begingroup$

      You can get it immediately:
      $$sum_{cyc}(a^3c-a^3b)=(a+b+c)(a-b)(b-c)(c-a)$$
      because for $a=b$ or $a=c$ or $b=c$ our determinant is equal to zero, which gives
      $$K(a+b+c)(a-b)(b-c)(c-a)$$ and it's enough to check the coefficient before $a^3c$, which is $1$, which gives $K=1$.






      share|cite|improve this answer









      $endgroup$





















        1












        $begingroup$

        Here is a geometrical interpretation of the formula :



        $$(a-b)(b-c)(c-a)(a+b+c).tag{1}$$



        I use the term "interpretation" because I have not written here a complete proof but rather an inductive way to obtain (1).



        Indeed, if the presence of factors $(a-b), (b-c), (c-a)$ look very natural (the determinant is zero if at has 2 identical columns), this is apparently not the case for factor $(a+b+c)$.



        Here is a context giving a natural interpretation to this factor.



        Reminder (see Theorem in https://proofwiki.org/wiki/Area_of_Triangle_in_Determinant_Form ) : Let $A,B,C$ be $3$ points in the plane.



        $$begin{vmatrix}1&1&1\x_A&x_B&x_C\y_A&y_B&y_Cend{vmatrix}=2 times area(ABC)$$



        (being understood that, is $A,B,C$ are all different, this determinant is zero iff $A,B,C$ are aligned).



        Here, we take points $A,B,C$ on the curve $Gamma$ with equation $y=x^3$. If they are distinct, they are aligned iff their abscissas are such that :



        $$x_A+x_B+x_C=0$$



        which is rather convincing when one looks at curve $Gamma$ (Fig. 1) :



        enter image description here



        Fig. 1 : the sum of abscissas of aligned points $A,B,C$ is $-1.5+0.5+1=0$.



        Now, the proof : the abscissas of intersection points of :



        $$begin{cases}y&=&x^3\y&=&ax+bend{cases} $$



        verify



        $$x^3-underbrace{0}_S x^2-ax-b=0.$$



        The sum of roots $S$ (coefficient of $-x^2$ according to Viète's formulas) is thus zero.






        share|cite|improve this answer











        $endgroup$













        • $begingroup$
          Wow. Thank you for your comment and [+1] for the unexpected geometric interpretation of the term $a+b+c$!
          $endgroup$
          – Song
          Mar 17 at 13:45



















        0












        $begingroup$

        1) By inspection : The determinant $= 0$ for $a=b$; $a=c$, and $b=c$ (Why?).



        Hence $pm (c-b)$, $pm(a-b)$, and $pm (a-c)$ are factors.



        You got $(c-b)(a-b)[a^2+ab -(c^2+cb)]$.



        Hence $pm (a-c)$ is a factor of $[a^2+ab -(c^2+cb)]$.



        $a^2-c^2+b(a-c)=$



        $ (a-c)(a+c)+b(a-c)=$



        $(a-c)(a+b+c).$






        share|cite|improve this answer









        $endgroup$














          Your Answer





          StackExchange.ifUsing("editor", function () {
          return StackExchange.using("mathjaxEditing", function () {
          StackExchange.MarkdownEditor.creationCallbacks.add(function (editor, postfix) {
          StackExchange.mathjaxEditing.prepareWmdForMathJax(editor, postfix, [["$", "$"], ["\\(","\\)"]]);
          });
          });
          }, "mathjax-editing");

          StackExchange.ready(function() {
          var channelOptions = {
          tags: "".split(" "),
          id: "69"
          };
          initTagRenderer("".split(" "), "".split(" "), channelOptions);

          StackExchange.using("externalEditor", function() {
          // Have to fire editor after snippets, if snippets enabled
          if (StackExchange.settings.snippets.snippetsEnabled) {
          StackExchange.using("snippets", function() {
          createEditor();
          });
          }
          else {
          createEditor();
          }
          });

          function createEditor() {
          StackExchange.prepareEditor({
          heartbeatType: 'answer',
          autoActivateHeartbeat: false,
          convertImagesToLinks: true,
          noModals: true,
          showLowRepImageUploadWarning: true,
          reputationToPostImages: 10,
          bindNavPrevention: true,
          postfix: "",
          imageUploader: {
          brandingHtml: "Powered by u003ca class="icon-imgur-white" href="https://imgur.com/"u003eu003c/au003e",
          contentPolicyHtml: "User contributions licensed under u003ca href="https://creativecommons.org/licenses/by-sa/3.0/"u003ecc by-sa 3.0 with attribution requiredu003c/au003e u003ca href="https://stackoverflow.com/legal/content-policy"u003e(content policy)u003c/au003e",
          allowUrls: true
          },
          noCode: true, onDemand: true,
          discardSelector: ".discard-answer"
          ,immediatelyShowMarkdownHelp:true
          });


          }
          });














          draft saved

          draft discarded


















          StackExchange.ready(
          function () {
          StackExchange.openid.initPostLogin('.new-post-login', 'https%3a%2f%2fmath.stackexchange.com%2fquestions%2f3150198%2fproving-left-beginsmallmatrix111-abc-a3b3c3-endsmallmatrix-rig%23new-answer', 'question_page');
          }
          );

          Post as a guest















          Required, but never shown

























          6 Answers
          6






          active

          oldest

          votes








          6 Answers
          6






          active

          oldest

          votes









          active

          oldest

          votes






          active

          oldest

          votes









          4












          $begingroup$

          Note that $(a^2+ab)-(c^2+cb)=(a-c)(a+c)+b(a-c)=(a-c)(a+b+c)$






          share|cite|improve this answer









          $endgroup$


















            4












            $begingroup$

            Note that $(a^2+ab)-(c^2+cb)=(a-c)(a+c)+b(a-c)=(a-c)(a+b+c)$






            share|cite|improve this answer









            $endgroup$
















              4












              4








              4





              $begingroup$

              Note that $(a^2+ab)-(c^2+cb)=(a-c)(a+c)+b(a-c)=(a-c)(a+b+c)$






              share|cite|improve this answer









              $endgroup$



              Note that $(a^2+ab)-(c^2+cb)=(a-c)(a+c)+b(a-c)=(a-c)(a+b+c)$







              share|cite|improve this answer












              share|cite|improve this answer



              share|cite|improve this answer










              answered Mar 16 at 9:11









              att eplatt epl

              27012




              27012























                  2












                  $begingroup$

                  Given a $4times 4$ Vandermonde matrix
                  $$
                  left[begin{matrix}
                  color{red}1&color{red}1&color{red}1&1\color{red}a&color{red}b&color{red}c&d\ a^2&b^2&c^2&color{blue}{d^2}\color{red}{a^3}&color{red}{b^3}&color{red}{c^3}&d^3
                  end{matrix}right],
                  $$
                  note that the desired determinant is the minor of $d^2$. Considering Laplace expansion, it appears as a minus(-) coefficient of $d^2$ in the Vandermonde determinant
                  $$begin{align*}
                  &color{red}{(b-a)(c-a)(c-b)}(d-a)(d-b)(d-c)\=&color{red}{(b-a)(c-a)(c-b)}(d^3-color{blue}{(a+b+c)}d^2+cdots).
                  end{align*}$$
                  Thus we obtain the desired determinant
                  $$color{red}{(b-a)(c-a)(c-b)}color{blue}{(a+b+c)}.
                  $$






                  share|cite|improve this answer











                  $endgroup$













                  • $begingroup$
                    [+1] very interesting.
                    $endgroup$
                    – Jean Marie
                    Mar 16 at 23:50










                  • $begingroup$
                    I just gave another way to consider this formula.
                    $endgroup$
                    – Jean Marie
                    Mar 17 at 13:34
















                  2












                  $begingroup$

                  Given a $4times 4$ Vandermonde matrix
                  $$
                  left[begin{matrix}
                  color{red}1&color{red}1&color{red}1&1\color{red}a&color{red}b&color{red}c&d\ a^2&b^2&c^2&color{blue}{d^2}\color{red}{a^3}&color{red}{b^3}&color{red}{c^3}&d^3
                  end{matrix}right],
                  $$
                  note that the desired determinant is the minor of $d^2$. Considering Laplace expansion, it appears as a minus(-) coefficient of $d^2$ in the Vandermonde determinant
                  $$begin{align*}
                  &color{red}{(b-a)(c-a)(c-b)}(d-a)(d-b)(d-c)\=&color{red}{(b-a)(c-a)(c-b)}(d^3-color{blue}{(a+b+c)}d^2+cdots).
                  end{align*}$$
                  Thus we obtain the desired determinant
                  $$color{red}{(b-a)(c-a)(c-b)}color{blue}{(a+b+c)}.
                  $$






                  share|cite|improve this answer











                  $endgroup$













                  • $begingroup$
                    [+1] very interesting.
                    $endgroup$
                    – Jean Marie
                    Mar 16 at 23:50










                  • $begingroup$
                    I just gave another way to consider this formula.
                    $endgroup$
                    – Jean Marie
                    Mar 17 at 13:34














                  2












                  2








                  2





                  $begingroup$

                  Given a $4times 4$ Vandermonde matrix
                  $$
                  left[begin{matrix}
                  color{red}1&color{red}1&color{red}1&1\color{red}a&color{red}b&color{red}c&d\ a^2&b^2&c^2&color{blue}{d^2}\color{red}{a^3}&color{red}{b^3}&color{red}{c^3}&d^3
                  end{matrix}right],
                  $$
                  note that the desired determinant is the minor of $d^2$. Considering Laplace expansion, it appears as a minus(-) coefficient of $d^2$ in the Vandermonde determinant
                  $$begin{align*}
                  &color{red}{(b-a)(c-a)(c-b)}(d-a)(d-b)(d-c)\=&color{red}{(b-a)(c-a)(c-b)}(d^3-color{blue}{(a+b+c)}d^2+cdots).
                  end{align*}$$
                  Thus we obtain the desired determinant
                  $$color{red}{(b-a)(c-a)(c-b)}color{blue}{(a+b+c)}.
                  $$






                  share|cite|improve this answer











                  $endgroup$



                  Given a $4times 4$ Vandermonde matrix
                  $$
                  left[begin{matrix}
                  color{red}1&color{red}1&color{red}1&1\color{red}a&color{red}b&color{red}c&d\ a^2&b^2&c^2&color{blue}{d^2}\color{red}{a^3}&color{red}{b^3}&color{red}{c^3}&d^3
                  end{matrix}right],
                  $$
                  note that the desired determinant is the minor of $d^2$. Considering Laplace expansion, it appears as a minus(-) coefficient of $d^2$ in the Vandermonde determinant
                  $$begin{align*}
                  &color{red}{(b-a)(c-a)(c-b)}(d-a)(d-b)(d-c)\=&color{red}{(b-a)(c-a)(c-b)}(d^3-color{blue}{(a+b+c)}d^2+cdots).
                  end{align*}$$
                  Thus we obtain the desired determinant
                  $$color{red}{(b-a)(c-a)(c-b)}color{blue}{(a+b+c)}.
                  $$







                  share|cite|improve this answer














                  share|cite|improve this answer



                  share|cite|improve this answer








                  edited Mar 16 at 9:53

























                  answered Mar 16 at 9:47









                  SongSong

                  18.5k21651




                  18.5k21651












                  • $begingroup$
                    [+1] very interesting.
                    $endgroup$
                    – Jean Marie
                    Mar 16 at 23:50










                  • $begingroup$
                    I just gave another way to consider this formula.
                    $endgroup$
                    – Jean Marie
                    Mar 17 at 13:34


















                  • $begingroup$
                    [+1] very interesting.
                    $endgroup$
                    – Jean Marie
                    Mar 16 at 23:50










                  • $begingroup$
                    I just gave another way to consider this formula.
                    $endgroup$
                    – Jean Marie
                    Mar 17 at 13:34
















                  $begingroup$
                  [+1] very interesting.
                  $endgroup$
                  – Jean Marie
                  Mar 16 at 23:50




                  $begingroup$
                  [+1] very interesting.
                  $endgroup$
                  – Jean Marie
                  Mar 16 at 23:50












                  $begingroup$
                  I just gave another way to consider this formula.
                  $endgroup$
                  – Jean Marie
                  Mar 17 at 13:34




                  $begingroup$
                  I just gave another way to consider this formula.
                  $endgroup$
                  – Jean Marie
                  Mar 17 at 13:34











                  1












                  $begingroup$

                  $$begin{vmatrix}1&1\c^2+cb+b^2&a^2+ab+b^2end{vmatrix}$$



                  $$=begin{vmatrix}1&1-1\c^2+cb+b^2&a^2+ab+b^2-(c^2+cb+b^2)end{vmatrix}$$



                  $$=begin{vmatrix}1&0\c^2+cb+b^2&a^2-c^2+b(a-c)end{vmatrix}$$



                  $$=a^2-c^2+b(a-c)$$



                  $$=(a-c)(a+c+b)$$






                  share|cite|improve this answer









                  $endgroup$


















                    1












                    $begingroup$

                    $$begin{vmatrix}1&1\c^2+cb+b^2&a^2+ab+b^2end{vmatrix}$$



                    $$=begin{vmatrix}1&1-1\c^2+cb+b^2&a^2+ab+b^2-(c^2+cb+b^2)end{vmatrix}$$



                    $$=begin{vmatrix}1&0\c^2+cb+b^2&a^2-c^2+b(a-c)end{vmatrix}$$



                    $$=a^2-c^2+b(a-c)$$



                    $$=(a-c)(a+c+b)$$






                    share|cite|improve this answer









                    $endgroup$
















                      1












                      1








                      1





                      $begingroup$

                      $$begin{vmatrix}1&1\c^2+cb+b^2&a^2+ab+b^2end{vmatrix}$$



                      $$=begin{vmatrix}1&1-1\c^2+cb+b^2&a^2+ab+b^2-(c^2+cb+b^2)end{vmatrix}$$



                      $$=begin{vmatrix}1&0\c^2+cb+b^2&a^2-c^2+b(a-c)end{vmatrix}$$



                      $$=a^2-c^2+b(a-c)$$



                      $$=(a-c)(a+c+b)$$






                      share|cite|improve this answer









                      $endgroup$



                      $$begin{vmatrix}1&1\c^2+cb+b^2&a^2+ab+b^2end{vmatrix}$$



                      $$=begin{vmatrix}1&1-1\c^2+cb+b^2&a^2+ab+b^2-(c^2+cb+b^2)end{vmatrix}$$



                      $$=begin{vmatrix}1&0\c^2+cb+b^2&a^2-c^2+b(a-c)end{vmatrix}$$



                      $$=a^2-c^2+b(a-c)$$



                      $$=(a-c)(a+c+b)$$







                      share|cite|improve this answer












                      share|cite|improve this answer



                      share|cite|improve this answer










                      answered Mar 16 at 8:38









                      lab bhattacharjeelab bhattacharjee

                      228k15158278




                      228k15158278























                          1












                          $begingroup$

                          You can get it immediately:
                          $$sum_{cyc}(a^3c-a^3b)=(a+b+c)(a-b)(b-c)(c-a)$$
                          because for $a=b$ or $a=c$ or $b=c$ our determinant is equal to zero, which gives
                          $$K(a+b+c)(a-b)(b-c)(c-a)$$ and it's enough to check the coefficient before $a^3c$, which is $1$, which gives $K=1$.






                          share|cite|improve this answer









                          $endgroup$


















                            1












                            $begingroup$

                            You can get it immediately:
                            $$sum_{cyc}(a^3c-a^3b)=(a+b+c)(a-b)(b-c)(c-a)$$
                            because for $a=b$ or $a=c$ or $b=c$ our determinant is equal to zero, which gives
                            $$K(a+b+c)(a-b)(b-c)(c-a)$$ and it's enough to check the coefficient before $a^3c$, which is $1$, which gives $K=1$.






                            share|cite|improve this answer









                            $endgroup$
















                              1












                              1








                              1





                              $begingroup$

                              You can get it immediately:
                              $$sum_{cyc}(a^3c-a^3b)=(a+b+c)(a-b)(b-c)(c-a)$$
                              because for $a=b$ or $a=c$ or $b=c$ our determinant is equal to zero, which gives
                              $$K(a+b+c)(a-b)(b-c)(c-a)$$ and it's enough to check the coefficient before $a^3c$, which is $1$, which gives $K=1$.






                              share|cite|improve this answer









                              $endgroup$



                              You can get it immediately:
                              $$sum_{cyc}(a^3c-a^3b)=(a+b+c)(a-b)(b-c)(c-a)$$
                              because for $a=b$ or $a=c$ or $b=c$ our determinant is equal to zero, which gives
                              $$K(a+b+c)(a-b)(b-c)(c-a)$$ and it's enough to check the coefficient before $a^3c$, which is $1$, which gives $K=1$.







                              share|cite|improve this answer












                              share|cite|improve this answer



                              share|cite|improve this answer










                              answered Mar 16 at 8:45









                              Michael RozenbergMichael Rozenberg

                              109k1896201




                              109k1896201























                                  1












                                  $begingroup$

                                  Here is a geometrical interpretation of the formula :



                                  $$(a-b)(b-c)(c-a)(a+b+c).tag{1}$$



                                  I use the term "interpretation" because I have not written here a complete proof but rather an inductive way to obtain (1).



                                  Indeed, if the presence of factors $(a-b), (b-c), (c-a)$ look very natural (the determinant is zero if at has 2 identical columns), this is apparently not the case for factor $(a+b+c)$.



                                  Here is a context giving a natural interpretation to this factor.



                                  Reminder (see Theorem in https://proofwiki.org/wiki/Area_of_Triangle_in_Determinant_Form ) : Let $A,B,C$ be $3$ points in the plane.



                                  $$begin{vmatrix}1&1&1\x_A&x_B&x_C\y_A&y_B&y_Cend{vmatrix}=2 times area(ABC)$$



                                  (being understood that, is $A,B,C$ are all different, this determinant is zero iff $A,B,C$ are aligned).



                                  Here, we take points $A,B,C$ on the curve $Gamma$ with equation $y=x^3$. If they are distinct, they are aligned iff their abscissas are such that :



                                  $$x_A+x_B+x_C=0$$



                                  which is rather convincing when one looks at curve $Gamma$ (Fig. 1) :



                                  enter image description here



                                  Fig. 1 : the sum of abscissas of aligned points $A,B,C$ is $-1.5+0.5+1=0$.



                                  Now, the proof : the abscissas of intersection points of :



                                  $$begin{cases}y&=&x^3\y&=&ax+bend{cases} $$



                                  verify



                                  $$x^3-underbrace{0}_S x^2-ax-b=0.$$



                                  The sum of roots $S$ (coefficient of $-x^2$ according to Viète's formulas) is thus zero.






                                  share|cite|improve this answer











                                  $endgroup$













                                  • $begingroup$
                                    Wow. Thank you for your comment and [+1] for the unexpected geometric interpretation of the term $a+b+c$!
                                    $endgroup$
                                    – Song
                                    Mar 17 at 13:45
















                                  1












                                  $begingroup$

                                  Here is a geometrical interpretation of the formula :



                                  $$(a-b)(b-c)(c-a)(a+b+c).tag{1}$$



                                  I use the term "interpretation" because I have not written here a complete proof but rather an inductive way to obtain (1).



                                  Indeed, if the presence of factors $(a-b), (b-c), (c-a)$ look very natural (the determinant is zero if at has 2 identical columns), this is apparently not the case for factor $(a+b+c)$.



                                  Here is a context giving a natural interpretation to this factor.



                                  Reminder (see Theorem in https://proofwiki.org/wiki/Area_of_Triangle_in_Determinant_Form ) : Let $A,B,C$ be $3$ points in the plane.



                                  $$begin{vmatrix}1&1&1\x_A&x_B&x_C\y_A&y_B&y_Cend{vmatrix}=2 times area(ABC)$$



                                  (being understood that, is $A,B,C$ are all different, this determinant is zero iff $A,B,C$ are aligned).



                                  Here, we take points $A,B,C$ on the curve $Gamma$ with equation $y=x^3$. If they are distinct, they are aligned iff their abscissas are such that :



                                  $$x_A+x_B+x_C=0$$



                                  which is rather convincing when one looks at curve $Gamma$ (Fig. 1) :



                                  enter image description here



                                  Fig. 1 : the sum of abscissas of aligned points $A,B,C$ is $-1.5+0.5+1=0$.



                                  Now, the proof : the abscissas of intersection points of :



                                  $$begin{cases}y&=&x^3\y&=&ax+bend{cases} $$



                                  verify



                                  $$x^3-underbrace{0}_S x^2-ax-b=0.$$



                                  The sum of roots $S$ (coefficient of $-x^2$ according to Viète's formulas) is thus zero.






                                  share|cite|improve this answer











                                  $endgroup$













                                  • $begingroup$
                                    Wow. Thank you for your comment and [+1] for the unexpected geometric interpretation of the term $a+b+c$!
                                    $endgroup$
                                    – Song
                                    Mar 17 at 13:45














                                  1












                                  1








                                  1





                                  $begingroup$

                                  Here is a geometrical interpretation of the formula :



                                  $$(a-b)(b-c)(c-a)(a+b+c).tag{1}$$



                                  I use the term "interpretation" because I have not written here a complete proof but rather an inductive way to obtain (1).



                                  Indeed, if the presence of factors $(a-b), (b-c), (c-a)$ look very natural (the determinant is zero if at has 2 identical columns), this is apparently not the case for factor $(a+b+c)$.



                                  Here is a context giving a natural interpretation to this factor.



                                  Reminder (see Theorem in https://proofwiki.org/wiki/Area_of_Triangle_in_Determinant_Form ) : Let $A,B,C$ be $3$ points in the plane.



                                  $$begin{vmatrix}1&1&1\x_A&x_B&x_C\y_A&y_B&y_Cend{vmatrix}=2 times area(ABC)$$



                                  (being understood that, is $A,B,C$ are all different, this determinant is zero iff $A,B,C$ are aligned).



                                  Here, we take points $A,B,C$ on the curve $Gamma$ with equation $y=x^3$. If they are distinct, they are aligned iff their abscissas are such that :



                                  $$x_A+x_B+x_C=0$$



                                  which is rather convincing when one looks at curve $Gamma$ (Fig. 1) :



                                  enter image description here



                                  Fig. 1 : the sum of abscissas of aligned points $A,B,C$ is $-1.5+0.5+1=0$.



                                  Now, the proof : the abscissas of intersection points of :



                                  $$begin{cases}y&=&x^3\y&=&ax+bend{cases} $$



                                  verify



                                  $$x^3-underbrace{0}_S x^2-ax-b=0.$$



                                  The sum of roots $S$ (coefficient of $-x^2$ according to Viète's formulas) is thus zero.






                                  share|cite|improve this answer











                                  $endgroup$



                                  Here is a geometrical interpretation of the formula :



                                  $$(a-b)(b-c)(c-a)(a+b+c).tag{1}$$



                                  I use the term "interpretation" because I have not written here a complete proof but rather an inductive way to obtain (1).



                                  Indeed, if the presence of factors $(a-b), (b-c), (c-a)$ look very natural (the determinant is zero if at has 2 identical columns), this is apparently not the case for factor $(a+b+c)$.



                                  Here is a context giving a natural interpretation to this factor.



                                  Reminder (see Theorem in https://proofwiki.org/wiki/Area_of_Triangle_in_Determinant_Form ) : Let $A,B,C$ be $3$ points in the plane.



                                  $$begin{vmatrix}1&1&1\x_A&x_B&x_C\y_A&y_B&y_Cend{vmatrix}=2 times area(ABC)$$



                                  (being understood that, is $A,B,C$ are all different, this determinant is zero iff $A,B,C$ are aligned).



                                  Here, we take points $A,B,C$ on the curve $Gamma$ with equation $y=x^3$. If they are distinct, they are aligned iff their abscissas are such that :



                                  $$x_A+x_B+x_C=0$$



                                  which is rather convincing when one looks at curve $Gamma$ (Fig. 1) :



                                  enter image description here



                                  Fig. 1 : the sum of abscissas of aligned points $A,B,C$ is $-1.5+0.5+1=0$.



                                  Now, the proof : the abscissas of intersection points of :



                                  $$begin{cases}y&=&x^3\y&=&ax+bend{cases} $$



                                  verify



                                  $$x^3-underbrace{0}_S x^2-ax-b=0.$$



                                  The sum of roots $S$ (coefficient of $-x^2$ according to Viète's formulas) is thus zero.







                                  share|cite|improve this answer














                                  share|cite|improve this answer



                                  share|cite|improve this answer








                                  edited Mar 17 at 13:33

























                                  answered Mar 17 at 10:55









                                  Jean MarieJean Marie

                                  31k42255




                                  31k42255












                                  • $begingroup$
                                    Wow. Thank you for your comment and [+1] for the unexpected geometric interpretation of the term $a+b+c$!
                                    $endgroup$
                                    – Song
                                    Mar 17 at 13:45


















                                  • $begingroup$
                                    Wow. Thank you for your comment and [+1] for the unexpected geometric interpretation of the term $a+b+c$!
                                    $endgroup$
                                    – Song
                                    Mar 17 at 13:45
















                                  $begingroup$
                                  Wow. Thank you for your comment and [+1] for the unexpected geometric interpretation of the term $a+b+c$!
                                  $endgroup$
                                  – Song
                                  Mar 17 at 13:45




                                  $begingroup$
                                  Wow. Thank you for your comment and [+1] for the unexpected geometric interpretation of the term $a+b+c$!
                                  $endgroup$
                                  – Song
                                  Mar 17 at 13:45











                                  0












                                  $begingroup$

                                  1) By inspection : The determinant $= 0$ for $a=b$; $a=c$, and $b=c$ (Why?).



                                  Hence $pm (c-b)$, $pm(a-b)$, and $pm (a-c)$ are factors.



                                  You got $(c-b)(a-b)[a^2+ab -(c^2+cb)]$.



                                  Hence $pm (a-c)$ is a factor of $[a^2+ab -(c^2+cb)]$.



                                  $a^2-c^2+b(a-c)=$



                                  $ (a-c)(a+c)+b(a-c)=$



                                  $(a-c)(a+b+c).$






                                  share|cite|improve this answer









                                  $endgroup$


















                                    0












                                    $begingroup$

                                    1) By inspection : The determinant $= 0$ for $a=b$; $a=c$, and $b=c$ (Why?).



                                    Hence $pm (c-b)$, $pm(a-b)$, and $pm (a-c)$ are factors.



                                    You got $(c-b)(a-b)[a^2+ab -(c^2+cb)]$.



                                    Hence $pm (a-c)$ is a factor of $[a^2+ab -(c^2+cb)]$.



                                    $a^2-c^2+b(a-c)=$



                                    $ (a-c)(a+c)+b(a-c)=$



                                    $(a-c)(a+b+c).$






                                    share|cite|improve this answer









                                    $endgroup$
















                                      0












                                      0








                                      0





                                      $begingroup$

                                      1) By inspection : The determinant $= 0$ for $a=b$; $a=c$, and $b=c$ (Why?).



                                      Hence $pm (c-b)$, $pm(a-b)$, and $pm (a-c)$ are factors.



                                      You got $(c-b)(a-b)[a^2+ab -(c^2+cb)]$.



                                      Hence $pm (a-c)$ is a factor of $[a^2+ab -(c^2+cb)]$.



                                      $a^2-c^2+b(a-c)=$



                                      $ (a-c)(a+c)+b(a-c)=$



                                      $(a-c)(a+b+c).$






                                      share|cite|improve this answer









                                      $endgroup$



                                      1) By inspection : The determinant $= 0$ for $a=b$; $a=c$, and $b=c$ (Why?).



                                      Hence $pm (c-b)$, $pm(a-b)$, and $pm (a-c)$ are factors.



                                      You got $(c-b)(a-b)[a^2+ab -(c^2+cb)]$.



                                      Hence $pm (a-c)$ is a factor of $[a^2+ab -(c^2+cb)]$.



                                      $a^2-c^2+b(a-c)=$



                                      $ (a-c)(a+c)+b(a-c)=$



                                      $(a-c)(a+b+c).$







                                      share|cite|improve this answer












                                      share|cite|improve this answer



                                      share|cite|improve this answer










                                      answered Mar 16 at 10:02









                                      Peter SzilasPeter Szilas

                                      11.6k2822




                                      11.6k2822






























                                          draft saved

                                          draft discarded




















































                                          Thanks for contributing an answer to Mathematics Stack Exchange!


                                          • Please be sure to answer the question. Provide details and share your research!

                                          But avoid



                                          • Asking for help, clarification, or responding to other answers.

                                          • Making statements based on opinion; back them up with references or personal experience.


                                          Use MathJax to format equations. MathJax reference.


                                          To learn more, see our tips on writing great answers.




                                          draft saved


                                          draft discarded














                                          StackExchange.ready(
                                          function () {
                                          StackExchange.openid.initPostLogin('.new-post-login', 'https%3a%2f%2fmath.stackexchange.com%2fquestions%2f3150198%2fproving-left-beginsmallmatrix111-abc-a3b3c3-endsmallmatrix-rig%23new-answer', 'question_page');
                                          }
                                          );

                                          Post as a guest















                                          Required, but never shown





















































                                          Required, but never shown














                                          Required, but never shown












                                          Required, but never shown







                                          Required, but never shown

































                                          Required, but never shown














                                          Required, but never shown












                                          Required, but never shown







                                          Required, but never shown







                                          Popular posts from this blog

                                          Magento 2 - Add success message with knockout Planned maintenance scheduled April 23, 2019 at 23:30 UTC (7:30pm US/Eastern) Announcing the arrival of Valued Associate #679: Cesar Manara Unicorn Meta Zoo #1: Why another podcast?Success / Error message on ajax request$.widget is not a function when loading a homepage after add custom jQuery on custom themeHow can bind jQuery to current document in Magento 2 When template load by ajaxRedirect page using plugin in Magento 2Magento 2 - Update quantity and totals of cart page without page reload?Magento 2: Quote data not loaded on knockout checkoutMagento 2 : I need to change add to cart success message after adding product into cart through pluginMagento 2.2.5 How to add additional products to cart from new checkout step?Magento 2 Add error/success message with knockoutCan't validate Post Code on checkout page

                                          Fil:Tokke komm.svg

                                          Where did Arya get these scars? Unicorn Meta Zoo #1: Why another podcast? Announcing the arrival of Valued Associate #679: Cesar Manara Favourite questions and answers from the 1st quarter of 2019Why did Arya refuse to end it?Has the pronunciation of Arya Stark's name changed?Has Arya forgiven people?Why did Arya Stark lose her vision?Why can Arya still use the faces?Has the Narrow Sea become narrower?Does Arya Stark know how to make poisons outside of the House of Black and White?Why did Nymeria leave Arya?Why did Arya not kill the Lannister soldiers she encountered in the Riverlands?What is the current canonical age of Sansa, Bran and Arya Stark?